- Sat Nov 30, 2013 12:00 am
#36510
Complete Question Explanation
Must Be True. The correct answer choice is (A)
The commentator in this stimulus notes that articles criticizing the environmental movement have
been appearing regularly in newspapers. The commentator introduces Winslow’s belief that this is
not the result of an anti-environmental bias among the media, but is due instead to editors’ preference
for “daring” articles that challenge prevailing political viewpoints.
The commentator agrees with Winslow regarding what drives newspapers to run articles that
challenge prevailing political orthodoxy. There is also disagreement with Winslow, however: The
commentator asserts that environmentalism is not politically orthodox, so anti-environmentalists are
not the rebels that they may portray themselves to be.
The question stem asks which answer choice is most strongly supported by the commentator’s
remarks. The correct answer choice will likely be a conclusion or an inference that can be logically
drawn from the stimulus.
Answer choice (A): This is the correct answer choice. The commentator does agree with Winslow
about newspaper editors’ preferences for controversial articles, apparently chosen in an effort to
appear daring.
Answer choice (B): In the last sentence of the stimulus, the commentator notes that the anti-environmentalists
have succeeded somewhat in selling themselves as renegades, so it would be
inaccurate, according to the information in the stimulus, to claim that the author doesn’t believe there
has been such successful self-promotion. This answer choice should therefore be eliminated.
Answer choice (C): Winslow’s explanation does provide reasons why such critiques are published
regularly, so this assertion is inaccurate, and this answer choice is incorrect.
Answer choice (D): The commentator would not agree with the assertion that the refuted position is
the prevailing political position, having stated that the refuted position is serious environmentalism,
which is not politically orthodox according to the commentator, so this answer choice should be
eliminated.
Answer choice (E): The commentator says that serious environmentalism is by no means politically
orthodox, but does not make the claim, or the implication, that it will eventually become a prevailing
political position. While the assertion in this answer choice could in time prove to be accurate, it is
not supported by the information in the stimulus, and this choice should be eliminated.
Must Be True. The correct answer choice is (A)
The commentator in this stimulus notes that articles criticizing the environmental movement have
been appearing regularly in newspapers. The commentator introduces Winslow’s belief that this is
not the result of an anti-environmental bias among the media, but is due instead to editors’ preference
for “daring” articles that challenge prevailing political viewpoints.
The commentator agrees with Winslow regarding what drives newspapers to run articles that
challenge prevailing political orthodoxy. There is also disagreement with Winslow, however: The
commentator asserts that environmentalism is not politically orthodox, so anti-environmentalists are
not the rebels that they may portray themselves to be.
The question stem asks which answer choice is most strongly supported by the commentator’s
remarks. The correct answer choice will likely be a conclusion or an inference that can be logically
drawn from the stimulus.
Answer choice (A): This is the correct answer choice. The commentator does agree with Winslow
about newspaper editors’ preferences for controversial articles, apparently chosen in an effort to
appear daring.
Answer choice (B): In the last sentence of the stimulus, the commentator notes that the anti-environmentalists
have succeeded somewhat in selling themselves as renegades, so it would be
inaccurate, according to the information in the stimulus, to claim that the author doesn’t believe there
has been such successful self-promotion. This answer choice should therefore be eliminated.
Answer choice (C): Winslow’s explanation does provide reasons why such critiques are published
regularly, so this assertion is inaccurate, and this answer choice is incorrect.
Answer choice (D): The commentator would not agree with the assertion that the refuted position is
the prevailing political position, having stated that the refuted position is serious environmentalism,
which is not politically orthodox according to the commentator, so this answer choice should be
eliminated.
Answer choice (E): The commentator says that serious environmentalism is by no means politically
orthodox, but does not make the claim, or the implication, that it will eventually become a prevailing
political position. While the assertion in this answer choice could in time prove to be accurate, it is
not supported by the information in the stimulus, and this choice should be eliminated.